Ask your own question, for FREE!
Mathematics 62 Online
OpenStudy (anonymous):

Determine whether the following sequence is increasing, decreasing, or not monotonic. Is the sequence bounded?

OpenStudy (anonymous):

(a) a_n = cos(n*pi / 2) (b) a_n = n/(n^2 + 1)

OpenStudy (anonymous):

first one i would just put in n = 1, 2, 3, and see what you get

OpenStudy (anonymous):

it is certainly bounded because cosine is bounded above by 1 and below by -1 did you try plugging in the numbers? do 4 and it will be very clear

OpenStudy (anonymous):

Oh I see, but what is the part b bounded by?

OpenStudy (anonymous):

I know it decreases but I still need to find the bounds.

OpenStudy (anonymous):

did you get 0,-1,0,1,0,-1,... for the first one?

OpenStudy (anonymous):

For the monotonic sequence theorem, does the sequence have to be bounded on BOTH sides? (top and bottom)?

OpenStudy (anonymous):

it is bounded below by 0 for sure

OpenStudy (anonymous):

wait are you talking about question 1 or 2?

OpenStudy (anonymous):

and yes I got that.... for part a. I got the same pattern.

OpenStudy (anonymous):

ok so on to part b

OpenStudy (anonymous):

What is part b bounded by? Yes ok

OpenStudy (anonymous):

you know the limit is zero right?

OpenStudy (anonymous):

yes

OpenStudy (anonymous):

That is only the lower bound. Don't we need to find an upper bound also?

OpenStudy (anonymous):

are you trying to show it is monotone and bounded?

OpenStudy (anonymous):

Yes^

OpenStudy (anonymous):

ok so it is bounded above by \(\frac{1}{2}\) but that is not important it is bounded below by 0 because the terms are all positive so what is left is to see if it is monotone decreasing

OpenStudy (anonymous):

and it does decrease. how did you receive the bounded above part?

OpenStudy (anonymous):

also does the sequence have to be bounded ABOVE and BELOW to be counted as bounded?

OpenStudy (anonymous):

i replaced n by 1 if it is decreasing it is largest when n is smallest

OpenStudy (anonymous):

n could be negative numbers though, correct?

OpenStudy (anonymous):

no

OpenStudy (anonymous):

????

OpenStudy (anonymous):

\(a_n\) n is a positive integer

OpenStudy (anonymous):

\[a_1,a_2,a_3,a_4,...\]

OpenStudy (anonymous):

otherwise nothing would be anything if you could go to either plus or minus infinity, what a mess that would be

OpenStudy (anonymous):

Sounds good, thanks

OpenStudy (anonymous):

the easiest way to show the sequence is monotone decreasing is the take the derivative of the corresponding function \[f(x)=\frac{x}{x^2+1}\] and show that if \(x>1\) the derivative is negative, so the function is decreasing and therefore so is the sequence

OpenStudy (anonymous):

otherwise it is very tedious to show it is decreasing, even though it is obvious

OpenStudy (anonymous):

I getcha, thanks

OpenStudy (anonymous):

yw

Can't find your answer? Make a FREE account and ask your own questions, OR help others and earn volunteer hours!

Join our real-time social learning platform and learn together with your friends!
Latest Questions
luvnickk: what typa music yall listen to ?
25 minutes ago 14 Replies 2 Medals
GothgirlLillian: Is music considered art?
1 hour ago 1 Reply 0 Medals
luvnickk: am newwww
3 hours ago 0 Replies 0 Medals
russianmafiya: can someone help me write a love song
4 hours ago 1 Reply 0 Medals
arrivhn: ADD ME ON DISCORD ICYAFFL
4 hours ago 4 Replies 1 Medal
arrivhn: whats is the accute of a cricle
4 hours ago 4 Replies 3 Medals
arrivhn: KNOCK KNOCK
4 hours ago 12 Replies 2 Medals
Can't find your answer? Make a FREE account and ask your own questions, OR help others and earn volunteer hours!

Join our real-time social learning platform and learn together with your friends!